LSAT and Law School Admissions Forum

Get expert LSAT preparation and law school admissions advice from PowerScore Test Preparation.

 Administrator
PowerScore Staff
  • PowerScore Staff
  • Posts: 8917
  • Joined: Feb 02, 2011
|
#22772
Complete Question Explanation

Must Be TrueX-SN. The correct answer choice is (A)

This stimulus consists of rules that can be diagramed as conditional statements.

UNLESS negotiations begin soon, the cease fire will be violated:
  • Violated→ Negotiations begin soon,
    Negotiations begin soon → Cease fire Will be violated.
Negotiations will be held ONLY IF other countries apply pressure:
  • Negotiations → Pressure
    PressureNegotiations.
An agreement will emerge ONLY IF pressure continues:
  • Agreement → Pressure continues
    continued pressure → agreement.
No negotiation held UNTIL troops demonstrate ability to enforce cease-fire:
  • Negotiations→ Troops demonstrate
    demonstrationnegotiations.
We can make many inferences; however, the question stem instructs us to pay attention to the case in which negotiations have begun. In that case, we know that there has been pressure, and that troops have demonstrated some ability to counter aggression.

Answer choice (A): This is the correct answer choice. Since this is a Mistaken Reversal of the first statement, this choice might not be true, which is a good answer to a Must Be True EXCEPT question.

Answer choice (B): This is definitely inferred, so this choice is wrong.

Answer choice (C): This is inferred, because the stimulus states that when the troops demonstrate ability to counter aggression, they suppress a major incentive for the two sides to resume hostilities, and we have already inferred that the troops have demonstrated that ability. This choice is incorrect.

Answer choice (D): This is definitely inferred, so this choice is wrong.

Answer choice (E): This is inferred, because it is a restatement of one of the conditions in the stimulus. Since the stimulus tells us that for negotiations to be successful, international pressure must continue, we know that international action IS NECESSSARY for resolution, or that resolution DEPENDS on international action. This choice is wrong.
 ellenb
  • Posts: 260
  • Joined: Oct 22, 2012
|
#11913
Dear Powerscore,

First of all, why did they not do a big chained diagram for this answer? Second of all, how does the statement "an agreement will emerge only if other countries continue such pressure throughout the negotations"

Agreement-->Continue Pressure

What part of the statement can we link it to? We just know that:

Negotiations will be held--> Other countries have pressured

Thus, one is the "continue pressure" necessary condition and the other is a necessary condition "other countries have pressured", so how does it relate to answer choice E?

There is nothing that helps us to infer Agreement-->Continue Pressure, we just know that negotiations will be held. But we cannot it link to the Agreement-->Continue Pressure statement.

Regards,

Ellen
 Steve Stein
PowerScore Staff
  • PowerScore Staff
  • Posts: 1153
  • Joined: Apr 11, 2011
|
#11927
Hi Ellen,

Unfortunately not every conditional reasoning question provides statements that can all be linked together in a big chain. In many cases the test makers seek to avoid being too straightforward.

The statement "agreement :arrow: continued pressure" cannot be easily linked to the other statements, which all deal with negotiations. Here are the diagrams:

..... ..... ..... ..... ..... ..... pressure from other countries
Cease fire :arrow: Negotiations :arrow: and
..... ..... ..... ..... ..... ..... show of force to suppress incentive to fight

Agreement :arrow: Continued pressure

Take a look at these diagrams, make sure you can see how they came together, and let me know whether they answer your question about answer choice (E).

Thanks!

~Steve
 ellenb
  • Posts: 260
  • Joined: Oct 22, 2012
|
#12198
I looked at the diagrams, so, why can we infer answer choice E, from the conditional statements. It seems that agreement -->pressure, is not linked to the rest of the stimulus, we just know that negotiations have been held, but since the agrrement-->pressure is separate by itself it does not affect our answer, we cannot conclude anything about an agreements since it is by itself and not connected to the rest of the stimulus.

Thanks

Ellen
 ellenb
  • Posts: 260
  • Joined: Oct 22, 2012
|
#14986
Dear Powerscore,

I did this question a little bit differently than it was done in the explanations,
I diagrammed the statements all in one big diagram. I will try to represent it the best I could, there should be some diagonal arrows, which I will try my best to represent.
---> Demonstrated Troops
Not Violated--> Negotiations Begin---> Pressure
Agreement-->


Thus, there are diagonal arrows from Negotiations Begin toward Demonstrated Troops.
And there is another diagonal arrow from Agreement towards pressure.

Thus, I connected all of this in a big diagram and I did not take the CP since I can do it in my head. Please let me know if this is a valid way of doing this, and whether I do not need to break it down line by line and come up with the CP for each statement. It seems this linking diagram method seems more efficient for the test and saves time.

Regards,

Ellen
 Nikki Siclunov
PowerScore Staff
  • PowerScore Staff
  • Posts: 1362
  • Joined: Aug 02, 2011
|
#14988
Hi Ellen,

From the looks of it, your diagram represents the conditional relationships correctly. And no, you don't need to list each one of them separately, along with their respective contrapositives. In fact, never do this myself because I don't have to: I know what each contrapositive is, and only use it if necessary for the creation of a conditional chain.

It's a much more efficient method, but it does require a fairly steep learning curve with a slightly higher probability of error. The benefits probably outweigh the costs, provided your accuracy on S/N questions has improved considerably.

Hope this helps! Let me know :-)
 hlc48@cornell.edu
  • Posts: 1
  • Joined: Jun 14, 2017
|
#36057
Hi-

I am confused why A is the correct answer.

The first statement: Unless negotiations begin soon, the cease-fire will be violated by one of the two sides to the dispute.

Diagram:
Cease-fire will not be violated ---> Negotiations begin soon

Thus, A states (the cease-fire has not been violated) - which must be true in order for the negotiations to be held....
Please clarify!
User avatar
 Dave Killoran
PowerScore Staff
  • PowerScore Staff
  • Posts: 5852
  • Joined: Mar 25, 2011
|
#36059
Hi HLC,

Thanks for the question! A few things here:

This is a MustX question, so the 4 wrong answers Must Be True and the correct answer will Not Necessarily Be True. With that in mind, and since we know the negotiations begin soon, must it be the case that the cease-fire has not been violated by either of the two sides (as A states)?

If the answer is yes, then this answer would be wrong. But, the answer is not yes, and the reason is that that making that inference would be a Mistaken Reversal. Here's how that works:

As you noted, the diagram to the first sentence is:

  • Violated→ Negotiations begin soon

The question stem then adds in the info that "Negotiations begin soon." This creates the following combination of premises:

  • Premise: Violated→ Negotiations begin soon

    Premise: Negotiations begin soon

Do those two premises result in the conclusion that Violated ? No, they do not. Consider the following example:

  • Premise: To get a 180 on the LSAT, you must study (180 :arrow: Study)

    Premise: Martin studied for the LSAT

Do the two premises above result in the conclusion that Martin will get a 180? No, he might get a 180 but it is not for sure. The reason we call it a Mistaken Reversal is that the relationship looks like this:

  • Premise: 180 :arrow: Study

    Premise: Study :arrow: 180(conclusion)

That's the same reversal arrangement with answer choice (A), and thus A cannot be concluded for sure. Since A is not necessarily the case, it is the correct answer.

Please let me know if that helps. Thanks!
 Alex3505
  • Posts: 10
  • Joined: Jun 20, 2017
|
#38021
Hi, I'm not logically understanding why A is the correct answer. I got this answer wrong initially choosing E as the correct answer by mistake. Based on my process of elimination once understanding answer E could be an extension of answer D , I was left with A as being the correct answer.
User avatar
 Dave Killoran
PowerScore Staff
  • PowerScore Staff
  • Posts: 5852
  • Joined: Mar 25, 2011
|
#38036
Hey Alex,

Did you happen to see my explanation of (A) on the prior page? It's the last explanation on the first page of the thread, and I show why is not necessarily true in the context of this argument.

Please check that out and let me know you thoughts. Thanks!

Get the most out of your LSAT Prep Plus subscription.

Analyze and track your performance with our Testing and Analytics Package.